Professor: Many introductory undergraduate science courses are intended to be "proving grounds," that is, they are de...

Ryan on October 24, 2019

Why is D correct? Why is C incorrect?

Thanks

Reply
Create a free account to read and take part in forum discussions.

Already have an account? log in

Skylar on October 27, 2019

@Ryan-Mahabir Happy to help!

The passage states that introductory science courses are designed to be "proving grounds" by only passing the students who are most committed to being science majors. The passage then claims that this intended purpose for "proving grounds" is not being met because some of the students who are least enthusiastic about science passed.

You may notice the switch in language from students who are/are not committed to being science majors to those who are/are not enthusiastic about science. How do we know that students can't be simultaneously committed to being science majors and unenthusiastic about science? This hits on the assumption we should be looking for in the answer choices.

(D) addresses the disparity between these two groups. It states that "none of the students in the very demanding introductory science courses who are least enthusiastic about science are among the students most committed to being science majors." This connects the two groups so that unenthusiastic students are also uncommitted, which allows the logic in the passage to make sense. If (D) was negated to say that some of the least enthusiastic students are uncommitted, then the logic in the passage would fall apart. Therefore, (D) is correct.

(C) states "some of the students in the very demanding introductory science courses who are most enthusiastic about science do not receive passing grades." However, since we have not made a connection between students being enthusiastic and being committed, we do not know that these enthusiastic students who did not pass are not also uncommitted to being science majors. If they were uncommitted, then the purpose of the "proving grounds" classes would still be met and the conclusion in the passage would be unsupported. Therefore, (C) is incorrect.

Does that make sense? Please let us know if you have any other questions!